URGENT PLS ANSWER QUICKLY​

URGENT PLS ANSWER QUICKLY

Answers

Answer 1

Answer:

9th is 44 mark me as brainlist

Answer 2

Answer:

Question 9 is perimeter


Related Questions

Hello, Brainly community!

This question is for all of those Calculus people out there.

The volume of a swimming pool is changing with respect to time, such that the volume is given by W(t), where W(t) is measured in cubic centimeters and t is measured in seconds. A tangent line is shown for W(t) at t = 3 seconds. Determine the best estimate for the value of the instantaneous rate of change of W(t) when t = 3.
(I've narrowed down the answer choices to 2, and just really need to find the right way of thinking to find the answer)

(A) W(lim t) as t goes to 3.
(B) [W(3.1) - W(2.9)] / 0.2.

Thank you in advance!

Answers

Answer:

(B)  [tex]\displaystyle \frac{W(3.1) - W(2.9)}{0.2}[/tex]

General Formulas and Concepts:

Calculus

Limits

Derivatives

The definition of a derivative is the slope of the tangent line.

Derivative Notation

Instantaneous Rates

Tangent Line: [tex]\displaystyle f'(x) = \frac{f(b) - f(a)}{b - a}[/tex]

Step-by-step explanation:

Since we are trying to find a rate at which W(t) changes, we must find the derivative at t = 3.

We are given 2 close answer choices that would have the same numerical answer but different meanings:

(A)  [tex]\displaystyle \lim_{t \to 3} W(t)[/tex](B)  [tex]\displaystyle \frac{W(3.1) - W(2.9)}{0.2}[/tex]

If we look at answer choice (A), we see that our units would simply just be volume. It would not have the units of a rate of change. Yes, it may be the closest numerically correct answer, but it does not tell us the rate at which the volume would be changing and it is not a derivative.

If we look at answer choice (B), we see that our units would be cm³/s, and that is most certainly a rate of change. Answer choice (B) is also a derivative at t = 3, and a derivative tells us what rate something is changing.

∴ Answer choice (B) will give us the best estimate for the value of the instantaneous rate of change of W(t) when t = 3.

Topic: AP Calculus AB/BC (Calculus I/I + II)

Unit: Differentiation

Book: College Calculus 10e

Graph the solution of the inequality 3/7(35x-14)<_ 21x/2+3

Answers

Answer:

You'll have a closed circle at x = 2, and shading to the left

See the diagram below

=========================================================

Explanation:

The fractions here are 3/7 and 21/2. The denominators of which are 7 and 2 respectively. The LCD is 7*2 = 14.

If we multiply both sides by 14, then this will clear out the denominators and make the fractions go away.

14*(3/7) = (14*3)/7 = 42/7 = 814*(21/2) = (14*21)/2 = 294/2 = 147

So if we multiplied both sides by 14, then we have these steps

[tex]\frac{3}{7}(35x-14) \le \frac{21x}{2}+3\\\\14*\frac{3}{7}(35x-14) \le 14*\left(\frac{21x}{2}+3\right)\\\\14*\frac{3}{7}(35x-14) \le 14*\left(\frac{21x}{2}\right)+14*\left(3\right)\\\\6(35x-14) \le 147x+42\\\\[/tex]

--------------------------

Let's isolate x

[tex]6(35x-14) \le 147x+42\\\\6(35x)+6(-14) \le 147x+42\\\\210x-84 \le 147x+42\\\\210x-147x \le 42+84\\\\63x \le 126\\\\x \le 126/63\\\\x \le 2\\\\[/tex]

The graph of this will consist of a closed or filled in circle at x = 2. We shade to the left to represent numbers smaller than 2.

So either x = 2 or x < 2.

If we used an open hole at 2, then we wouldn't be including 2 (but we want to include this endpoint).

See the diagram below.

What is the area of a cross section that is parallel to face BFGC?

Answers

Answer: 216cm2

Step-by-step explanation: 36x6

Which of the following recursive formulas represents the same arithmetic sequence as the explicit formula an = 5+ (n-1)2?

Answers

Answer:

[tex]a_1 = 5[/tex]

[tex]a_n = a_{n-1} + 2[/tex]

Step-by-step explanation:

Given

[tex]a_n = 5 + (n - 1)2[/tex]

Required

The equivalent recursive function

The general explicit function is:

[tex]a_n = a_1+ (n - 1)d[/tex]

So, by comparison

[tex]a_1 = 5[/tex]

[tex]d = 2[/tex]

The recursion of an arithmetic sequence is:

[tex]a_n = a_{n-1} + d[/tex]

Substitute 2 for d

[tex]a_n = a_{n-1} + 2[/tex]

Hence: (a) is correct

what do you understand by socialization​

Answers

Socialization is the lifelong process through which people learn the values and norms of a given society.

A line with a slope of 3 passes through the point (-1, 2).

Write an equation for this line in point-slope form.

Answers

Answer:

Step-by-step explanation:

Slope = m = 3

(x₁ , y₁) = (-1 , 2)

Point slope form: y - y₁ = m(x - x₁)

y - 2 = 3(x - [-1] )

y - 2 = 3(x + 1)

y - 2 = 3*x + 3*1

y - 2 = 3x + 3

    y = 3x + 3 + 2

    y = 3x + 5

How many black berry trees were sampled to create this graph

Answers

Answer:

C. 31 trees

Step-by-step explanation:

Add up all the frequencies: 3+3+8+10+5+2=31

Hope this helps!!!

Answer:

It's C. 31 trees

Step-by-step explanation:

That's about all I can say

Drag


the yellow point until an accurate "height" of the triangle is drawn. Afterwards, fill out the empty boxes below to determine the area of the triangle.

Answers

Answer:

I don't see a yellow point

Shirley buys fiction books for $20 each, and then marks up by 25% to
resell. What is the markup in dollars?

Answers

Answer:

$5

Step-by-step explanation:

Find the markup by finding 25% of 20:

20(0.25)

= 5

So, the markup is $5

**Does anybody knows how to do it***

instructions: state what additional information is required in order to know that the triangle are congruent for the given reason.

Given: ASA

Answers

With ASA, we need two angles and an included side. An included side is one that is between the two angles.

But, we need to make sure the sides match up. So, triangle KJI is congruent to triangle ZYX.

Therefore, we need to know that side JI is congruent to YX.

Hope this helps!

CAN SOMEBODY PLEASE HELP MEEEE

Answers

Answer:

105.

Step-by-step explanation:

.

These points are linear.
Find the slope.
x-3 -2 -1 0 1/2
y-3 -2 -1 0 1 2
slope = [?]

Answers

Answer:

Slope = 4

Step-by-step explanation:

The slope of a linear equation can be calculated as;

m = y2-y1/x2-x1

What we have to do here is to select any two points ;

(x1,y1) = (3,8)

(x2,y2) = (6,20)

m = (20-8)/(6-3) = 12/3 = 4

Which number completes the system of linear
inequalities represented by the graph?
y> 2x – 2 and x + 4y > _____

Answers

The system of linear inequalties that is represented by the graph is formed by the expressions y > 2 · x - 2 and x + 4 · y > - 12.

What system of inequalities generate the graph seen in the image?

Herein we have two inequalities generated by two linear functions whose form is described below:

f(x, y) > a (blue region)      (1)

g(x, y) > b (red region)      (2)

By direct comparison, we find that the limit of the red region is represented by the inequality y > 2 · x - 2. From the equation of the line we can derive an expression for the limit of the blue region:

Slope

m = [- 4 - (- 3)]/(4 - 0)

m = - 1/4

Intercept

b = y - m · x

b = - 3 - (- 1/4) · 0

b = - 3

Then, the inequation is represented by the linear equation:

y > - (1/4) · x - 3

4 · y > - x - 12

x + 4 · y > - 12

The system of linear inequalties that is represented by the graph is formed by the expressions y > 2 · x - 2 and x + 4 · y > - 12.

To learn more on inequalities: https://brainly.com/question/20383699

#SPJ1

Quadrilateral A B C D is shown. The uppercase right angle, angle A, is 79 degrees.
What are the remaining angle measures if the figure is to be a parallelogram?

m∠B =
°

m∠C =
°

m∠D =
°

Answers

Answer:

m∠B =  

✔ 101

°

m∠C =  

✔ 79

°

m∠D =  

✔ 101

°

Step-by-step explanation:

Answer:

The answer above is right!

The correct answers are:

First box: option C. 101

Second box: option B. 79

Third box: option C. 101

Step-by-step explanation:

Just got it right on edge - Hope it helps :)

Brainliest would be greatly appreciated :D

a bag contains 3 white balls and 4 red balls. A.second bag contains 4 white balls and 3 red balls. If a bag is picked at random from each bag. Find the probability that both are white

Answers

Answer:

12/49 chance that they are both white

Step-by-step explanation:

We have 2 events going on:

The first bag that contains: 3 white, 4 red

The second bag that contains: 4 white, 3 red

First let's find the probability of picking a white ball from each individual bag (not combing the chances yet):

For the 1st bag, there are 3 white balls out of 7 balls (add 3 and 4).

That means there's a 3/7 chance you'll get a white ball from the 1st bag.

For the 2nd bag, there are 4 white balls out of 7 balls (add 4 and 3).

That means there's a 4/7 chance you'll get a white ball from the 2nd bag.

To find the probability of getting both white from both of the bags, multiply 3/7 & 4/7:

3/7 * 4/7 = 12/49

You can't simplify 12/49 further, so that's your answer!

Hope it helps (●'◡'●)

Answer:

2/6

Step-by-step explanation:

_______________________________hope it helps________

the answer is... 2/6

find the gradients of line a and b

Answers

Answer:

Gradient of A: 2

Gradient of B: -1

Step-by-step explanation:

Gradient = change in y/change in x

✔️Gradient of A using two points on line A, (2, 5) and (0, 1):

Gradient = (1 - 5)/(0 - 2) = -4/-2

Simplify

Gradient of A = 2

✔️Gradient of B using two points on line B, (0, 5) and (5, 0):

Gradient = (0 - 5)/(5 - 0) = -5/5

Simplify

Gradient of B = -1

Help!!!
Find the domain of the function. Write the answer in interval notation.

Answers

Answer:

A

Step-by-step explanation:

Given

f(x) = [tex]\frac{2}{3x+2}[/tex]

The denominator cannot be zero as this would make f(x) undefined.

Equating the denominator to zero and solving gives the value that x cannot be.

3x + 2 = 0 ⇒ 3x = - 2 ⇒ x = - [tex]\frac{2}{3}[/tex] ← excluded value

Then

domain is ( - ∞ , - [tex]\frac{2}{3}[/tex] ) U ( - [tex]\frac{2}{3}[/tex], ∞ ) → A

Which of the following is not a way to represent the solution of the inequality 2(x − 1) greater than or equal to −12? (1 point)

x greater than or equal to −5
x less than or equal to −5
−5 less than or equal to x
A number line with a closed circle on negative 5 and shading to the right

Answers

Answer:

x less than or equal to -5

Step-by-step explanation:

2(x-1) ≥ -12

2x - 2 ≥ -12

2x ≥ -10

x ≥ -5

NEED HELPPPP RNNNNNNNN​

Answers

put it together first so
x+12-5x=3 solve from there it easy

If anyone can help with this ill mark Brainly

Answers

Answer:

In this case, we can do substitution.

Step-by-step explanation:

For the first one, (s - t)(x) = ((x - 5) - 4x^2)(x) = x^2 - 21x

For the second one, (s*t)(x) = ((x - 5) *4x^2)(x) = 4x^4 - 20x^3

And for the last one,  (s+t)(-2) = ((x - 5) + 4x^2)(-2) = -8x^2 - 2x + 10

Hope your happy with the answer :)

Ava’s gross pay is $2,500 per month. Her deductions include the following:

Federal income tax $290

State income tax $85

Social Security $112

Medicare $35

What is Ava’s net pay each month?

Answers

$1,978

Explanation:

Gross pay is what’s made before taxes and necessary deductions, net pay is after taxes and necessary deductions. So we minus the deductions to find the net pay. The equation is 2,500 - 290 - 85 - 112 - 35 = ? . The answer is $1,978.

I hope this helps. Please mark me the Brainliest, it’s not necessary but I put time and effort into every answer and I would appreciate it greatly. Have a great day, stay safe and stay healthy ! :)

Which is the area between the x-axis and y=x from x=1 to x=5

Answers

Answer:

[tex]\displaystyle A = 12[/tex]

General Formulas and Concepts:

Pre-Algebra

Order of Operations: BPEMDAS

BracketsParenthesisExponentsMultiplicationDivisionAdditionSubtractionLeft to Right

Algebra I

FunctionsFunction NotationGraphing

Calculus

Integrals

Definite IntegralsArea under the curve

Integration Rule [Reverse Power Rule]:                                                               [tex]\displaystyle \int {x^n} \, dx = \frac{x^{n + 1}}{n + 1} + C[/tex]

Integration Rule [Fundamental Theorem of Calculus 1]:                                     [tex]\displaystyle \int\limits^b_a {f(x)} \, dx = F(b) - F(a)[/tex]

Area of a Region Formula:                                                                                     [tex]\displaystyle A = \int\limits^b_a {[f(x) - g(x)]} \, dx[/tex]

Step-by-step explanation:

Step 1: Define

Identify

y = x

Interval: x = 1 to x = 5

Step 2: Sort

Graph the function. See Attachment.

Bounds of Integration: [1, 5]

Step 3: Find Area

Substitute in variables [Area of a Region Formula]:                                   [tex]\displaystyle A = \int\limits^5_1 {x} \, dx[/tex][Integral] Integrate [Integration Rule - Reverse Power Rule]:                     [tex]\displaystyle A = \frac{x^2}{2} \bigg| \limits^5_1[/tex]Evaluate [Integration Rule - Fundamental Theorem of Calculus 1]:           [tex]\displaystyle A = 12[/tex]

Topic: AP Calculus AB/BC (Calculus I/I + II)

Unit: Integration

Book: College Calculus 10e

Identify the parts of the following algebraic expression.

-8z + 1
2
y - 7.7

Term:
Variable:
Coefficient:
Constant:

Answers

Answer:

-8z+1

term:2

variable:z

coefficient:-8

constant:1

2

term:1

variable:nil

coefficient:nil

constant:2

y-7.7

term:2

variable:y

coefficient:nil

constant:-7.7

The solution is given below.

What is number?

A number is a mathematical object used to count, measure, and label. The original examples are the natural numbers 1, 2, 3, 4, and so forth. Numbers can be represented in language with number words.

now, we get,

-8z+1

term:2

variable: z

coefficient:-8

constant:1

again,

2

term:1

variable : nil

coefficient : nil

constant:2

now,

y-7.7

term:2

variable : y

coefficient : nil

constant:-7.7

To learn more on number click:

brainly.com/question/17429689

#SPJ2

Question: Dentify the parts of the following algebraic expression.

-8z + 1

2

y - 7.7

Term:

Variable:

Coefficient:

Constant:

PLS HELP ASAP, I need it in 10 mins. I GIVE 15 PTS !!!! if v1 = (3,-4) and v2 = (2,6) then v1*v2 is equal to which of the following?
A. 30
B. (-12, -24)
C. (6,-24)
D. -18

Answers

It is C. Good luck on ur test!!!!

v₁·v₂ is -18 which is correct option(D)

What are Arithmetic operations?

Arithmetic operations can also be specified by the subtract, divide, and multiply built-in functions.

The operator that perform arithmetic operation are called arithmetic operators .

Operators which let do basic mathematical calculations

+ Addition operation : Adds values on either side of the operator.

For example 4 + 2 = 6

- Subtraction operation : Subtracts right hand operand from left hand operand.

for example 4 -2 = 2

* Multiplication operation : Multiplies values on either side of the operator

For example 4*2 = 8

/ Division operation : Divides left hand operand by right hand operand

For example 4/2 = 2

Given that,

v1 = (3,-4) and v2 = (2,6)

To determine v₁·v₂

v₁·v₂ = 3·(2) - 4·(6)

v₁·v₂ = 6 - 24

v₁·v₂ = -18

Hence, the v₁·v₂ is -18.

Learn more about Arithmetic operations here:

brainly.com/question/25834626

#SPJ2

Can someone help with problems fivethroughseven

Answers

Answer:

5) 15120

6) 11880

7) 336

Step-by-step explanation:

The formula for permutation where mPn is m!/(m-n)!

Applying this to question 5, we get 9!/4!, which is 15120.

For question 6, we get 12!/8!, which is 11880.

For question 7, we get 8!/5!, which is 336.

Find the equation of a line that is perpendicular to y = -3x – 1 and passes through the point
(3,2).
Give your answer in the form y = mx + b.

Answers

the answer is:
y = 1/3x + 1
I think the answer is y= ⅓ + 1

the red line below is perpendicular to which of the following

Answers

The red line is perpendicular to the y-axis.

Find the value of x from the following given figures.​

Answers

solution :-

here,

We know that interior opposite angles are equal.

So,

110° = 50° + x (being interior opposite angles)

110° - 50° = x

60° = x

the value of x =60°

hope it is helpful to you ☺️

Solve for the questions (both of them) and label you answers for which question

Answers

Jjbbh I don’t under stand

PLZ HELP its due soon!!!

Answers

Answer:

E)

Step-by-step explanation:

1) 6.50 (starting with his current wage)

2) 6.50 + 0.25 = 6.75 $

3) 6.75 + 0.25 = 7.00 $

4) 7.00 + 0.25 = 7.25 $

5) 7.25 + 0.25 = 7.50 $

6) 7.50 + 0.25 = 7.75 $

Other Questions
cual es el tamao de partcula del alcohol PLS HELP I WILL GIVE BRANLIEST JUST PLS QUICKK Charles has five 1 pound bags of different color sands. For an art project, he will use 3/8 pounds of each bag of sand to create a colorful sand art jar. How much sand will be in Charles's sand art jar? I NEED HELP FAST!!!!!(05.03 MC) John reads an equal number of pages of a book every week. The graph below shows the number of pages of the book left to read, y, after x weeks: Which equation best models the relationship between x and y?A. y= -5x+25B. y= -25x+125C. y= -25x+175D. y= -5x+125 How did technological advancements change manufacturing methods duringthe Industrial Revolution? [tex]what \: is \: reflection \: of \: light \: \: \: \: {?}[/tex] By ____________________ Andy Kirk means, do the (viewers') need engage with this work, or is it entirely voluntary A gymnast falls from a height onto a trampoline. For a moment, both the gymnasts kinetic energy and gravitational potential energy are zero. How is the gymnasts mechanical energy stored for that moment? Question 12 options: rest energy chemical energy elastic energy thermal energy True or False- The thermosphere contains no water vapor. Zimmerman argues that natural resources are functional discuss. Make a conjecture about the diagram below. Is AC greater than, less than, or equal to BC? Explain your reasoning. Give structural formula of ethylene. What type of bond is found in between carbon and hydrogen? Why is the bond between carbon and carbon weak in it? Over the last year, Calzone Corporation paid a quarterly dividend of $0.10 in each of the four quarters. The current stock price of Calzone Corporation is $39.78. What is the dividend yield for Calzone stock (-3).(+9)-(-24)-(+6).(+2) write a letter to your best friend to apologize for your behavior and explain what really happened. please help On Mar 3, Lyons Company paid dividends of $1,000. Use your knowledge of what a correct journal entry should look like to identify what would be include helppppp meeeeeeee plzzzzzzzz PLEASE HELLPP!!! Choose the best graph that represents the linear equation:-x = 2y + 1Graph AOn a coordinate plane, a line goes through (negative 1, 0) and (1, negative 1).Graph BOn a coordinate plane, a line goes through (negative 3, negative 1) and (1, 1).Graph COn a coordinate plane, a line goes through (1, 0) and (5, negative 2).Graph DOn a coordinate plane, a line goes through (negative 3, negative 2) and (1, 0).a.Graph Ac.Graph Cb.Graph Bd.Graph DPlease select the best answer from the choices providedABCD 1.a machine gun fires a ball with an initial velocity of 600m/s with an elevation of 30 with respect to the ground neglecting air resistance calculate: a.the maximum height that can be reached? b.the time of flight of the bullet?c.the maximum horizontal displacement of the ired bullet? Read the statements below, then answer the question that follows.1. Taking a Spanish class to learn the structure of the Spanish language.2. Repeating what a parent says after listening to them speak Spanish.3. Watching an older sibling throw a basketball through a hoop.4. Attempting to imitate a sibling by throwing a basketball through a hoop. Surface Area of PrismsCan someone show me how to solve and also what the formula is. Please!